Difference between revisions of "1997 AIME Problems/Problem 6"
(a third solution) |
m (→Solution 3) |
||
(3 intermediate revisions by 2 users not shown) | |||
Line 18: | Line 18: | ||
(m-6)(n-6) &=& 36 | (m-6)(n-6) &=& 36 | ||
\end{eqnarray*}</cmath> | \end{eqnarray*}</cmath> | ||
− | Clearly <math>n</math> is maximized when <math>m = 7, n = \boxed{ | + | Clearly <math>n</math> is maximized when <math>m = 7, n = \boxed{042}</math>. |
== Solution 2 == | == Solution 2 == | ||
Line 28: | Line 28: | ||
With this restriction of <math>m>6</math>, the larger <math>m</math> gets, the smaller the fraction <math>\frac{6m}{m-6}</math> becomes. This can be proven either by calculus, by noting that <math>n = \frac{6m}{m-6}</math> is a transformed hyperbola, or by dividing out the rational function to get <math>n = 6 + \frac{36}{m - 6}.</math> | With this restriction of <math>m>6</math>, the larger <math>m</math> gets, the smaller the fraction <math>\frac{6m}{m-6}</math> becomes. This can be proven either by calculus, by noting that <math>n = \frac{6m}{m-6}</math> is a transformed hyperbola, or by dividing out the rational function to get <math>n = 6 + \frac{36}{m - 6}.</math> | ||
− | Either way, minimizng <math>m</math> will maximize <math>n</math>, and the smallest integer <math>m</math> such that <math>n</math> is positive is <math>m=7</math>, giving <math>n = \boxed{ | + | Either way, minimizng <math>m</math> will maximize <math>n</math>, and the smallest integer <math>m</math> such that <math>n</math> is positive is <math>m=7</math>, giving <math>n = \boxed{042}</math> |
== Solution 3 == | == Solution 3 == | ||
− | From the formula for the measure for an individual angle of a regular n-gon, <math>180 - \frac{360}{n}</math>, the measure of <math>\angle A_2A_1A_n = 180 - \frac{360}{n}</math>. Together with the fact that an equilateral triangle has angles measuring 60 degrees, the measure of <math>\angle A_nA_1B = 120 + \frac{360}{n}</math> (Notice that this value decreases as <math>n</math> increases; hence, we are looking for the least possible value of <math>\angle A_nA_1B</math>). For <math>A_n, A_1, B</math> to be vertices of a regular polygon, <math>\angle A_nA_1B</math> must be of the form <math>180 - \frac{360}{n}</math>, where <math>n</math> is a natural number greater than or equal to 3. It is obvious that <math>\angle A_nA_1B > 120</math>. The least angle satisfying this condition is <math>180 - \frac{360}{7}</math>. Equating this with <math>120 + \frac{360}{n}</math> and solving yields <math>n = \boxed{ | + | From the formula for the measure for an individual angle of a regular n-gon, <math>180 - \frac{360}{n}</math>, the measure of <math>\angle A_2A_1A_n = 180 - \frac{360}{n}</math>. Together with the fact that an equilateral triangle has angles measuring 60 degrees, the measure of <math>\angle A_nA_1B = 120 + \frac{360}{n}</math> (Notice that this value decreases as <math>n</math> increases; hence, we are looking for the least possible value of <math>\angle A_nA_1B</math>). For <math>A_n, A_1, B</math> to be vertices of a regular polygon, <math>\angle A_nA_1B</math> must be of the form <math>180 - \frac{360}{n}</math>, where <math>n</math> is a natural number greater than or equal to 3. It is obvious that <math>\angle A_nA_1B > 120</math>. The least angle satisfying this condition is <math>180 - \frac{360}{7}</math>. Equating this with <math>120 + \frac{360}{n}</math> and solving yields <math>n = \boxed{042}</math> |
== See also == | == See also == | ||
Line 39: | Line 39: | ||
[[Category:Intermediate Algebra Problems]] | [[Category:Intermediate Algebra Problems]] | ||
[[Category:Intermediate Geometry Problems]] | [[Category:Intermediate Geometry Problems]] | ||
+ | {{MAA Notice}} |
Latest revision as of 23:39, 21 December 2018
Problem
Point is in the exterior of the regular -sided polygon , and is an equilateral triangle. What is the largest value of for which , , and are consecutive vertices of a regular polygon?
Solution 1
Let the other regular polygon have sides. Using the interior angle of a regular polygon formula, we have , , and . Since those three angles add up to ,
Using SFFT,
Clearly is maximized when .
Solution 2
As above, find that using the formula for the interior angle of a polygon.
Solve for to find that . Clearly, for to be positive.
With this restriction of , the larger gets, the smaller the fraction becomes. This can be proven either by calculus, by noting that is a transformed hyperbola, or by dividing out the rational function to get
Either way, minimizng will maximize , and the smallest integer such that is positive is , giving
Solution 3
From the formula for the measure for an individual angle of a regular n-gon, , the measure of . Together with the fact that an equilateral triangle has angles measuring 60 degrees, the measure of (Notice that this value decreases as increases; hence, we are looking for the least possible value of ). For to be vertices of a regular polygon, must be of the form , where is a natural number greater than or equal to 3. It is obvious that . The least angle satisfying this condition is . Equating this with and solving yields
See also
1997 AIME (Problems • Answer Key • Resources) | ||
Preceded by Problem 5 |
Followed by Problem 7 | |
1 • 2 • 3 • 4 • 5 • 6 • 7 • 8 • 9 • 10 • 11 • 12 • 13 • 14 • 15 | ||
All AIME Problems and Solutions |
The problems on this page are copyrighted by the Mathematical Association of America's American Mathematics Competitions.